Difference between revisions of "1985 AJHSME Problem 2"
(→Solution 2) |
(→Solution 1) |
||
Line 8: | Line 8: | ||
We can add as follows: | We can add as follows: | ||
<cmath>90+91+92+93+94+95+96+97+98+99= 10(90) +1+2+3+4+5+6+7+8+9 = 900 + 45 = \boxed{945}</cmath> | <cmath>90+91+92+93+94+95+96+97+98+99= 10(90) +1+2+3+4+5+6+7+8+9 = 900 + 45 = \boxed{945}</cmath> | ||
− | The answer is <math>\ | + | The answer is <math></math>\boxed{\textbf{(B)}\ 945}$. |
==Solution 2== | ==Solution 2== |
Revision as of 08:35, 22 March 2025
Problem
Solution 1
We can add as follows:
The answer is $$ (Error compiling LaTeX. Unknown error_msg)\boxed{\textbf{(B)}\ 945}$.
Solution 2
Pair the numbers like so:
The sum of each pair is
and there are
pairs, so the sum is
and the answer is
.
Cheap Solution
We know that and
Quick estimation reveals that this sum is in between these two numbers, so the only answer available is
Solution 3
You see that is equal to
. You can use the formula
to get 45+900=945
Video Solution
~savannahsolver